subject
Mathematics, 10.04.2021 04:10 officialrogerfp3gf2s

HELP PLEASE What are the vertices of the resulting image A'B'C'D'E' after rotating the figure 90° about the origin? Drag numbers to complete the coordinates. Numbers may be used once, more than once, or not at all.

A' ( ? , ? ), B' ( ? , ? ), C' ( ?, ? ), D' ( ? , ? ), E' ( ? , ? )

Possible numbers: -6, -4, -2, 0, 2, 4, 6.​


HELP PLEEEEASE

What are the vertices of the resulting image A'B'C'D'E' after rotating the fig

ansver
Answers: 1

Another question on Mathematics

question
Mathematics, 21.06.2019 14:30
Geometry: determine whether segment mn is parallel to segment kl. justify your answer. jm 6 mk 3 jn 8 nl 4
Answers: 1
question
Mathematics, 21.06.2019 15:30
Find the gradient of f(x,y,z)equals=left parenthesis x squared plus y squared plus z squared right parenthesis superscript negative 1 divided by 2 baseline plus ln left parenthesis x right parenthesis x2+y2+z2−1/2+ln(xyz) at the point left parenthesis negative 2 comma 1 comma negative 2 right parenthesis(−2,1,−2).
Answers: 1
question
Mathematics, 21.06.2019 19:30
The measure of an angle is 117°. what is the measure of a supplementary angle?
Answers: 2
question
Mathematics, 21.06.2019 21:30
In a test for esp (extrasensory perception), the experimenter looks at cards that are hidden from the subject. each card contains either a star, a circle, a wave, a cross or a square.(five shapes) as the experimenter looks at each of 20 cards in turn, the subject names the shape on the card. when the esp study described above discovers a subject whose performance appears to be better than guessing, the study continues at greater length. the experimenter looks at many cards bearing one of five shapes (star, square, circle, wave, and cross) in an order determined by random numbers. the subject cannot see the experimenter as he looks at each card in turn, in order to avoid any possible nonverbal clues. the answers of a subject who does not have esp should be independent observations, each with probability 1/5 of success. we record 1000 attempts. which of the following assumptions must be met in order to solve this problem? it's reasonable to assume normality 0.8(1000), 0.2(1000)%30 approximately normal 0.8(1000), 0.2(1000)% 10 approximately normal srs it is reasonable to assume the total number of cards is over 10,000 it is reasonable to assume the total number of cards is over 1000
Answers: 1
You know the right answer?
HELP PLEASE What are the vertices of the resulting image A'B'C'D'E' after rotating the figure 90° a...
Questions
Questions on the website: 13722367